Quantcast
  • Register
PhysicsOverflow is a next-generation academic platform for physicists and astronomers, including a community peer review system and a postgraduate-level discussion forum analogous to MathOverflow.

Welcome to PhysicsOverflow! PhysicsOverflow is an open platform for community peer review and graduate-level Physics discussion.

Please help promote PhysicsOverflow ads elsewhere if you like it.

News

PO is now at the Physics Department of Bielefeld University!

New printer friendly PO pages!

Migration to Bielefeld University was successful!

Please vote for this year's PhysicsOverflow ads!

Please do help out in categorising submissions. Submit a paper to PhysicsOverflow!

... see more

Tools for paper authors

Submit paper
Claim Paper Authorship

Tools for SE users

Search User
Reclaim SE Account
Request Account Merger
Nativise imported posts
Claim post (deleted users)
Import SE post

Users whose questions have been imported from Physics Stack Exchange, Theoretical Physics Stack Exchange, or any other Stack Exchange site are kindly requested to reclaim their account and not to register as a new user.

Public \(\beta\) tools

Report a bug with a feature
Request a new functionality
404 page design
Send feedback

Attributions

(propose a free ad)

Site Statistics

205 submissions , 163 unreviewed
5,047 questions , 2,200 unanswered
5,345 answers , 22,709 comments
1,470 users with positive rep
816 active unimported users
More ...

  Unstable particles and quantum field theory

+ 4 like - 0 dislike
1065 views

I am searching for not too old literature on the quantum description of unstable particles. I am referring to something beyond the ad-hoc S-matrix description based on the optical theorem common to textbooks such as those given by Peskin and Schröder or Weinberg etc. The book "Open Quantum Systems and Feynman Integrals" by Exner seems to go in this direction. But I find the formulation there very mathematical. Certainly it is possible to understand it but I am worried if I will get the connection to physics.


This post imported from StackExchange Physics at 2015-08-01 13:46 (UTC), posted by SE-user highsciguy

asked Nov 17, 2012 in Theoretical Physics by highsciguy (20 points) [ revision history ]
edited Aug 1, 2015 by Dilaton
Possibly related: physics.stackexchange.com/q/39287/2451

This post imported from StackExchange Physics at 2015-08-01 13:46 (UTC), posted by SE-user Qmechanic

1 Answer

+ 1 like - 0 dislike

In relativistic QFT, unstable particles are defined by poles in the correlation functions, analytically continued to the second sheet.

Actual computations are usually done using Kadanoff-Baym equations, using the CTP (closed time pathy) formalism. See, e.g., http://arxiv.org/pdf/0801.4324

This post imported from StackExchange Physics at 2015-08-01 13:47 (UTC), posted by SE-user Arnold Neumaier
answered Nov 18, 2012 by Arnold Neumaier (15,787 points) [ no revision ]
Thank you. I am familiar with the Kadanoff-Baym equations. I am

This post imported from StackExchange Physics at 2015-08-01 13:47 (UTC), posted by SE-user highsciguy
... merely searching for the connection to conventional quantum field theory and the optical theorem as well as physical interpretations.

This post imported from StackExchange Physics at 2015-08-01 13:47 (UTC), posted by SE-user highsciguy
@highsciguy: The connection is given on the usual heuristic level by the CTP functional integral. - What physical interpretation are you looking for beyond knowing that the K-B equations give a dynamics for the correlation functions, hence tell something about the unstable particle spectrum?

This post imported from StackExchange Physics at 2015-08-01 13:47 (UTC), posted by SE-user Arnold Neumaier
Concerning physical interpretations I think of modified decay laws, Quantum Zeno effect and so on. Things which are usually discussed for individual particles. How may such effects be extracted from the statistical Kadanoff-Baym description or how can I understand it in terms of ordinary relativistic quantum mechanics.

This post imported from StackExchange Physics at 2015-08-01 13:47 (UTC), posted by SE-user highsciguy
@highsciguy: I haven't seen a single paper on work on the measurement problem based on nonequilibrium relativistic QFT. Thus this looks like (difficult) unexplored territory. The quasiparticle content depends on the state, and measurement should probably be modelled in terms of suitable initial conditions and external forces. But the questions asked in the two communities are too different to relate them easily. - For the interpretation of unstable particles (resonances) as poles of the S-matrix in ordinary QM, see, e.g., Vol. 3 of Thirrings course on mathematical physics.

This post imported from StackExchange Physics at 2015-08-01 13:47 (UTC), posted by SE-user Arnold Neumaier

Your answer

Please use answers only to (at least partly) answer questions. To comment, discuss, or ask for clarification, leave a comment instead.
To mask links under text, please type your text, highlight it, and click the "link" button. You can then enter your link URL.
Please consult the FAQ for as to how to format your post.
This is the answer box; if you want to write a comment instead, please use the 'add comment' button.
Live preview (may slow down editor)   Preview
Your name to display (optional):
Privacy: Your email address will only be used for sending these notifications.
Anti-spam verification:
If you are a human please identify the position of the character covered by the symbol $\varnothing$ in the following word:
p$\varnothing$ysicsOverflow
Then drag the red bullet below over the corresponding character of our banner. When you drop it there, the bullet changes to green (on slow internet connections after a few seconds).
Please complete the anti-spam verification




user contributions licensed under cc by-sa 3.0 with attribution required

Your rights
...